Calculus Help Finding the Arc Length












1












$begingroup$


So I'm having trouble with this problem:




Let $$x=frac{y^2+2y}{8}-ln(y+1)$$
Find the arclength for $0leq yleq 2$.




My work. I know the Arc length formula is $(1+ (x'^2))^{1/2}$ in this case but when I take $x'$ I get $$x' = (y/4 + 1/4 - 1/(y+1))$$ and when I square that I get something way too hectic and adding $1$ to all of that makes it even more of a mess. Am I missing something here?










share|cite|improve this question











$endgroup$












  • $begingroup$
    $ln3+1$ comes from what Robert Z suggests
    $endgroup$
    – Sameer Baheti
    Dec 17 '18 at 6:38
















1












$begingroup$


So I'm having trouble with this problem:




Let $$x=frac{y^2+2y}{8}-ln(y+1)$$
Find the arclength for $0leq yleq 2$.




My work. I know the Arc length formula is $(1+ (x'^2))^{1/2}$ in this case but when I take $x'$ I get $$x' = (y/4 + 1/4 - 1/(y+1))$$ and when I square that I get something way too hectic and adding $1$ to all of that makes it even more of a mess. Am I missing something here?










share|cite|improve this question











$endgroup$












  • $begingroup$
    $ln3+1$ comes from what Robert Z suggests
    $endgroup$
    – Sameer Baheti
    Dec 17 '18 at 6:38














1












1








1





$begingroup$


So I'm having trouble with this problem:




Let $$x=frac{y^2+2y}{8}-ln(y+1)$$
Find the arclength for $0leq yleq 2$.




My work. I know the Arc length formula is $(1+ (x'^2))^{1/2}$ in this case but when I take $x'$ I get $$x' = (y/4 + 1/4 - 1/(y+1))$$ and when I square that I get something way too hectic and adding $1$ to all of that makes it even more of a mess. Am I missing something here?










share|cite|improve this question











$endgroup$




So I'm having trouble with this problem:




Let $$x=frac{y^2+2y}{8}-ln(y+1)$$
Find the arclength for $0leq yleq 2$.




My work. I know the Arc length formula is $(1+ (x'^2))^{1/2}$ in this case but when I take $x'$ I get $$x' = (y/4 + 1/4 - 1/(y+1))$$ and when I square that I get something way too hectic and adding $1$ to all of that makes it even more of a mess. Am I missing something here?







calculus integration definite-integrals arc-length






share|cite|improve this question















share|cite|improve this question













share|cite|improve this question




share|cite|improve this question








edited Dec 17 '18 at 7:20









Martin Rosenau

1,1661310




1,1661310










asked Dec 17 '18 at 5:53









krauser126krauser126

474




474












  • $begingroup$
    $ln3+1$ comes from what Robert Z suggests
    $endgroup$
    – Sameer Baheti
    Dec 17 '18 at 6:38


















  • $begingroup$
    $ln3+1$ comes from what Robert Z suggests
    $endgroup$
    – Sameer Baheti
    Dec 17 '18 at 6:38
















$begingroup$
$ln3+1$ comes from what Robert Z suggests
$endgroup$
– Sameer Baheti
Dec 17 '18 at 6:38




$begingroup$
$ln3+1$ comes from what Robert Z suggests
$endgroup$
– Sameer Baheti
Dec 17 '18 at 6:38










1 Answer
1






active

oldest

votes


















1












$begingroup$

You are going in the right direction. However note that after squaring and adding $1$ a perfect square turns out:
$$1+(x'(y))^2=1+left(frac{y+1}{4}-frac{1}{y+1}right)^2
=frac{(y+1)^2}{16}+frac{1}{(y+1)^2}+frac{1}{2}=left(frac{t}{4}+frac{1}{t}right)^2.$$

where $t=y+1$. Now integrate the square root of the right-hand side for $t$ in $(1,3)$ and you will get the result.






share|cite|improve this answer











$endgroup$













  • $begingroup$
    So what is the final result?
    $endgroup$
    – Robert Z
    Dec 17 '18 at 6:45










  • $begingroup$
    Thank you. I will try it now. I was just wondering, is there a method in order to figure out how to factor this into a perfect square? (completing the square doesn't work here so how did you manage to figure it out?
    $endgroup$
    – krauser126
    Dec 17 '18 at 6:48










  • $begingroup$
    "completing the square doesn't work here" why?
    $endgroup$
    – Robert Z
    Dec 17 '18 at 6:49










  • $begingroup$
    Wouldn't that only work in something of the form Ax^2 + bx + c? Also, I got 1 + ln(3) as the answer.
    $endgroup$
    – krauser126
    Dec 17 '18 at 6:50










  • $begingroup$
    @krauser126 Use $(a+b)^2 = a^2 + b^2 + 2ab$. In this case $ab=frac14$
    $endgroup$
    – Dylan
    Dec 17 '18 at 6:52













Your Answer





StackExchange.ifUsing("editor", function () {
return StackExchange.using("mathjaxEditing", function () {
StackExchange.MarkdownEditor.creationCallbacks.add(function (editor, postfix) {
StackExchange.mathjaxEditing.prepareWmdForMathJax(editor, postfix, [["$", "$"], ["\\(","\\)"]]);
});
});
}, "mathjax-editing");

StackExchange.ready(function() {
var channelOptions = {
tags: "".split(" "),
id: "69"
};
initTagRenderer("".split(" "), "".split(" "), channelOptions);

StackExchange.using("externalEditor", function() {
// Have to fire editor after snippets, if snippets enabled
if (StackExchange.settings.snippets.snippetsEnabled) {
StackExchange.using("snippets", function() {
createEditor();
});
}
else {
createEditor();
}
});

function createEditor() {
StackExchange.prepareEditor({
heartbeatType: 'answer',
autoActivateHeartbeat: false,
convertImagesToLinks: true,
noModals: true,
showLowRepImageUploadWarning: true,
reputationToPostImages: 10,
bindNavPrevention: true,
postfix: "",
imageUploader: {
brandingHtml: "Powered by u003ca class="icon-imgur-white" href="https://imgur.com/"u003eu003c/au003e",
contentPolicyHtml: "User contributions licensed under u003ca href="https://creativecommons.org/licenses/by-sa/3.0/"u003ecc by-sa 3.0 with attribution requiredu003c/au003e u003ca href="https://stackoverflow.com/legal/content-policy"u003e(content policy)u003c/au003e",
allowUrls: true
},
noCode: true, onDemand: true,
discardSelector: ".discard-answer"
,immediatelyShowMarkdownHelp:true
});


}
});














draft saved

draft discarded


















StackExchange.ready(
function () {
StackExchange.openid.initPostLogin('.new-post-login', 'https%3a%2f%2fmath.stackexchange.com%2fquestions%2f3043581%2fcalculus-help-finding-the-arc-length%23new-answer', 'question_page');
}
);

Post as a guest















Required, but never shown

























1 Answer
1






active

oldest

votes








1 Answer
1






active

oldest

votes









active

oldest

votes






active

oldest

votes









1












$begingroup$

You are going in the right direction. However note that after squaring and adding $1$ a perfect square turns out:
$$1+(x'(y))^2=1+left(frac{y+1}{4}-frac{1}{y+1}right)^2
=frac{(y+1)^2}{16}+frac{1}{(y+1)^2}+frac{1}{2}=left(frac{t}{4}+frac{1}{t}right)^2.$$

where $t=y+1$. Now integrate the square root of the right-hand side for $t$ in $(1,3)$ and you will get the result.






share|cite|improve this answer











$endgroup$













  • $begingroup$
    So what is the final result?
    $endgroup$
    – Robert Z
    Dec 17 '18 at 6:45










  • $begingroup$
    Thank you. I will try it now. I was just wondering, is there a method in order to figure out how to factor this into a perfect square? (completing the square doesn't work here so how did you manage to figure it out?
    $endgroup$
    – krauser126
    Dec 17 '18 at 6:48










  • $begingroup$
    "completing the square doesn't work here" why?
    $endgroup$
    – Robert Z
    Dec 17 '18 at 6:49










  • $begingroup$
    Wouldn't that only work in something of the form Ax^2 + bx + c? Also, I got 1 + ln(3) as the answer.
    $endgroup$
    – krauser126
    Dec 17 '18 at 6:50










  • $begingroup$
    @krauser126 Use $(a+b)^2 = a^2 + b^2 + 2ab$. In this case $ab=frac14$
    $endgroup$
    – Dylan
    Dec 17 '18 at 6:52


















1












$begingroup$

You are going in the right direction. However note that after squaring and adding $1$ a perfect square turns out:
$$1+(x'(y))^2=1+left(frac{y+1}{4}-frac{1}{y+1}right)^2
=frac{(y+1)^2}{16}+frac{1}{(y+1)^2}+frac{1}{2}=left(frac{t}{4}+frac{1}{t}right)^2.$$

where $t=y+1$. Now integrate the square root of the right-hand side for $t$ in $(1,3)$ and you will get the result.






share|cite|improve this answer











$endgroup$













  • $begingroup$
    So what is the final result?
    $endgroup$
    – Robert Z
    Dec 17 '18 at 6:45










  • $begingroup$
    Thank you. I will try it now. I was just wondering, is there a method in order to figure out how to factor this into a perfect square? (completing the square doesn't work here so how did you manage to figure it out?
    $endgroup$
    – krauser126
    Dec 17 '18 at 6:48










  • $begingroup$
    "completing the square doesn't work here" why?
    $endgroup$
    – Robert Z
    Dec 17 '18 at 6:49










  • $begingroup$
    Wouldn't that only work in something of the form Ax^2 + bx + c? Also, I got 1 + ln(3) as the answer.
    $endgroup$
    – krauser126
    Dec 17 '18 at 6:50










  • $begingroup$
    @krauser126 Use $(a+b)^2 = a^2 + b^2 + 2ab$. In this case $ab=frac14$
    $endgroup$
    – Dylan
    Dec 17 '18 at 6:52
















1












1








1





$begingroup$

You are going in the right direction. However note that after squaring and adding $1$ a perfect square turns out:
$$1+(x'(y))^2=1+left(frac{y+1}{4}-frac{1}{y+1}right)^2
=frac{(y+1)^2}{16}+frac{1}{(y+1)^2}+frac{1}{2}=left(frac{t}{4}+frac{1}{t}right)^2.$$

where $t=y+1$. Now integrate the square root of the right-hand side for $t$ in $(1,3)$ and you will get the result.






share|cite|improve this answer











$endgroup$



You are going in the right direction. However note that after squaring and adding $1$ a perfect square turns out:
$$1+(x'(y))^2=1+left(frac{y+1}{4}-frac{1}{y+1}right)^2
=frac{(y+1)^2}{16}+frac{1}{(y+1)^2}+frac{1}{2}=left(frac{t}{4}+frac{1}{t}right)^2.$$

where $t=y+1$. Now integrate the square root of the right-hand side for $t$ in $(1,3)$ and you will get the result.







share|cite|improve this answer














share|cite|improve this answer



share|cite|improve this answer








edited Dec 17 '18 at 6:36

























answered Dec 17 '18 at 6:15









Robert ZRobert Z

98.6k1068139




98.6k1068139












  • $begingroup$
    So what is the final result?
    $endgroup$
    – Robert Z
    Dec 17 '18 at 6:45










  • $begingroup$
    Thank you. I will try it now. I was just wondering, is there a method in order to figure out how to factor this into a perfect square? (completing the square doesn't work here so how did you manage to figure it out?
    $endgroup$
    – krauser126
    Dec 17 '18 at 6:48










  • $begingroup$
    "completing the square doesn't work here" why?
    $endgroup$
    – Robert Z
    Dec 17 '18 at 6:49










  • $begingroup$
    Wouldn't that only work in something of the form Ax^2 + bx + c? Also, I got 1 + ln(3) as the answer.
    $endgroup$
    – krauser126
    Dec 17 '18 at 6:50










  • $begingroup$
    @krauser126 Use $(a+b)^2 = a^2 + b^2 + 2ab$. In this case $ab=frac14$
    $endgroup$
    – Dylan
    Dec 17 '18 at 6:52




















  • $begingroup$
    So what is the final result?
    $endgroup$
    – Robert Z
    Dec 17 '18 at 6:45










  • $begingroup$
    Thank you. I will try it now. I was just wondering, is there a method in order to figure out how to factor this into a perfect square? (completing the square doesn't work here so how did you manage to figure it out?
    $endgroup$
    – krauser126
    Dec 17 '18 at 6:48










  • $begingroup$
    "completing the square doesn't work here" why?
    $endgroup$
    – Robert Z
    Dec 17 '18 at 6:49










  • $begingroup$
    Wouldn't that only work in something of the form Ax^2 + bx + c? Also, I got 1 + ln(3) as the answer.
    $endgroup$
    – krauser126
    Dec 17 '18 at 6:50










  • $begingroup$
    @krauser126 Use $(a+b)^2 = a^2 + b^2 + 2ab$. In this case $ab=frac14$
    $endgroup$
    – Dylan
    Dec 17 '18 at 6:52


















$begingroup$
So what is the final result?
$endgroup$
– Robert Z
Dec 17 '18 at 6:45




$begingroup$
So what is the final result?
$endgroup$
– Robert Z
Dec 17 '18 at 6:45












$begingroup$
Thank you. I will try it now. I was just wondering, is there a method in order to figure out how to factor this into a perfect square? (completing the square doesn't work here so how did you manage to figure it out?
$endgroup$
– krauser126
Dec 17 '18 at 6:48




$begingroup$
Thank you. I will try it now. I was just wondering, is there a method in order to figure out how to factor this into a perfect square? (completing the square doesn't work here so how did you manage to figure it out?
$endgroup$
– krauser126
Dec 17 '18 at 6:48












$begingroup$
"completing the square doesn't work here" why?
$endgroup$
– Robert Z
Dec 17 '18 at 6:49




$begingroup$
"completing the square doesn't work here" why?
$endgroup$
– Robert Z
Dec 17 '18 at 6:49












$begingroup$
Wouldn't that only work in something of the form Ax^2 + bx + c? Also, I got 1 + ln(3) as the answer.
$endgroup$
– krauser126
Dec 17 '18 at 6:50




$begingroup$
Wouldn't that only work in something of the form Ax^2 + bx + c? Also, I got 1 + ln(3) as the answer.
$endgroup$
– krauser126
Dec 17 '18 at 6:50












$begingroup$
@krauser126 Use $(a+b)^2 = a^2 + b^2 + 2ab$. In this case $ab=frac14$
$endgroup$
– Dylan
Dec 17 '18 at 6:52






$begingroup$
@krauser126 Use $(a+b)^2 = a^2 + b^2 + 2ab$. In this case $ab=frac14$
$endgroup$
– Dylan
Dec 17 '18 at 6:52




















draft saved

draft discarded




















































Thanks for contributing an answer to Mathematics Stack Exchange!


  • Please be sure to answer the question. Provide details and share your research!

But avoid



  • Asking for help, clarification, or responding to other answers.

  • Making statements based on opinion; back them up with references or personal experience.


Use MathJax to format equations. MathJax reference.


To learn more, see our tips on writing great answers.




draft saved


draft discarded














StackExchange.ready(
function () {
StackExchange.openid.initPostLogin('.new-post-login', 'https%3a%2f%2fmath.stackexchange.com%2fquestions%2f3043581%2fcalculus-help-finding-the-arc-length%23new-answer', 'question_page');
}
);

Post as a guest















Required, but never shown





















































Required, but never shown














Required, but never shown












Required, but never shown







Required, but never shown

































Required, but never shown














Required, but never shown












Required, but never shown







Required, but never shown







Popular posts from this blog

How do I know what Microsoft account the skydrive app is syncing to?

When does type information flow backwards in C++?

Grease: Live!